View Kamer

00aa post

AI ECN 2019 

  1. Topic: Causes of Acute Mitral Regurgitation

Correct Answer: B. Hypertrophic and obstructive cardiomyopathy

Explanation: Acute mitral regurgitation is commonly caused by conditions that suddenly damage the mitral valve or its supporting structures, such as chordae tendineae rupture (rope breakage), trauma, bacterial endocarditis, or myocardial infarction. Hypertrophic and obstructive cardiomyopathy, while it can lead to chronic mitral regurgitation due to obstruction and abnormal valve motion, is not a typical cause of acute mitral regurgitation.

  1. Topic: Complications of Osler’s Endocarditis on Mitral Prolapse

Correct Answer: A. Multiple pulmonary septic embolisms

Explanation: Osler’s endocarditis, particularly when associated with mitral valve prolapse, can lead to complications such as right hemiplegia with aphasia due to embolic strokes, microscopic hematuria from emboli to the kidneys, lumbar spondylitis, and acute limb ischemia from arterial emboli. However, multiple pulmonary septic embolisms are more commonly associated with right-sided endocarditis rather than mitral prolapse, making option A the correct exception.

  1. Topic: Signs of Cardiac Tamponade

Correct Answer: C. Signs of pulmonary edema

Explanation: Cardiac tamponade is characterized by signs such as jugular venous distension (turgor of the jugular veins), a slightly mobile cardiac silhouette on fluoroscopy, a paradoxical pulse (pulse paradox), and a decrease in systolic blood pressure during inspiration. These are all indicative of the increased pressure on the heart and restricted cardiac filling. Signs of pulmonary edema, however, are not typically associated with cardiac tamponade and are more related to left heart failure, making option C the correct exception.

  1. Topic: Medications in Congestive Heart Failure

Correct Answer: C. Beta blockers

Explanation: While beta blockers are generally beneficial in managing chronic heart failure, they are strictly contraindicated in acute decompensated heart failure because they can reduce cardiac output and worsen symptoms. Other medications listed, such as digitalics, nitro derivatives, diuretics, and prazosin, are commonly used in managing congestive heart failure, especially during acute episodes.

  1. Topic: Malignant Arterial Hypertension

Correct Answer: A. Stage 2 fundus

Explanation: Malignant arterial hypertension is a severe form of hypertension characterized by very high diastolic blood pressure (often above 130 mmHg), signs of left ventricular failure, significant neurosensory disturbances, and renal insufficiency. However, it is associated with Stage 4 hypertensive retinopathy (not Stage 2), which includes papilledema, hemorrhages, and exudates.

  1. Topic: Risk Factors for Digoxin Toxicity

Correct Answer: C. Hyperthyroidism

Explanation: Digoxin toxicity is more likely in conditions such as kidney failure, very advanced heart failure, hyperkalemia, and hypercalcemia because these conditions affect digoxin’s metabolism and its effects on the heart. However, hyperthyroidism typically increases the metabolism of digoxin, potentially reducing its effect, rather than increasing the risk of toxicity.

  1. Topic: Paraneoplastic Syndromes in Bronchial Cancer

Correct Answer: E. Enophthalmos

Explanation: Paraneoplastic syndromes commonly associated with bronchial cancer include Schwartz-Bartter syndrome (SIADH), digital clubbing, Cushing’s syndrome, and hypercalcemia. These syndromes result from the ectopic production of hormones or cytokines by the tumor. Enophthalmos, which refers to the posterior displacement of the eye within the orbit, is not typically associated with bronchial cancer and is not a paraneoplastic syndrome.

  1. Topic: Diagnosis of Chronic Bronchitis

Correct Answer: A. Through interrogation

Explanation: Chronic bronchitis is primarily diagnosed based on clinical history, which involves questioning the patient (interrogation). The diagnosis is typically established when a patient reports a productive cough lasting for at least three months in two consecutive years. Although additional tests like respiratory functional exploration or imaging may help assess the extent of lung damage, the initial diagnosis hinges on the patient’s history.

  1. Topic: Complications of Bronchial Dilations

Correct Answer: D. Cystic fibrosis

Explanation: Cystic fibrosis is a genetic disorder that leads to bronchiectasis (bronchial dilations) but is not a complication of bronchial dilations. The other options, such as lung abscess, hemoptysis, purulent pleurisy, and chronic cor pulmonale, are recognized complications that can arise from bronchial dilations.

  1. Topic: Pre-capillary Pulmonary Arterial Hypertension

Correct Answer: A. Vasoconstriction due to hypoxia

Explanation: Pre-capillary pulmonary arterial hypertension in the context of chronic respiratory failure is primarily associated with vasoconstriction due to hypoxia. Hypoxia causes the pulmonary arteries to constrict, leading to increased pulmonary arterial pressure. The other options do not directly explain the mechanism behind pre-capillary pulmonary hypertension in this context.

  1. Topic: Diffuse Interstitial Pulmonary Fibrosis

Correct Answer: C. Left cancer

Explanation: Diffuse interstitial pulmonary fibrosis is characterized by clinical signs such as clubbing, dyspnea, weight loss, and crackles. However, “left cancer” is not associated with this condition. The clinical manifestations of diffuse interstitial pulmonary fibrosis are related to lung fibrosis and its effects, rather than to malignancies.

  1. Topic: Hemoptysis in a Former Tuberculosis Patient

Correct Answer: E. Bronchial candidiasis

Explanation: In a former tuberculosis patient experiencing hemoptysis, the following conditions are relevant: bronchial cancer, recurrence of pulmonary tuberculosis, parascaricidal dilatation of the bronchi, and aspergillosis as a sequel of tuberculosis. Bronchial candidiasis is not a common condition associated with hemoptysis in this context.

  1. Topic: Gastroesophageal Reflux

Correct Answer: A. It is secondary to retrograde esophageal contractions

Explanation: Gastroesophageal reflux disease (GERD) is primarily related to the incompetence of the lower esophageal sphincter, not retrograde esophageal contractions. It can occur with or without hiatal hernia, is often marked by heartburn, may present as pseudoanginal pain, and is associated with lower esophageal sphincter incontinence.

  1. Topic: Pyloric Stenosis in Young Adults

Correct Answer: C. Hypertrophy of the pylorus

Explanation: In young adults, the most common cause of a clinical syndrome resembling pyloric stenosis is hypertrophy of the pylorus. Gastric cancer, duodenal ulcer, and gastric ulcers are less commonly associated with this syndrome in this age group.

  1. Topic: Assessment of Upper Gastrointestinal Hemorrhage

Correct Answer: C. The number of units of blood and fluids infused for hemodynamic maintenance

Explanation: The most precise criterion for assessing the importance of an upper gastrointestinal hemorrhage is the number of units of blood and fluids infused for hemodynamic maintenance. This measure provides a direct indication of the volume of blood lost and the extent of resuscitation required. Other options such as hematocrit, the volume of blood aspirated, drop in blood pressure, and initial central venous pressure are important but less directly related to quantifying the blood loss and necessary intervention.

  1. Topic: Epigastric Pain Diagnosis

Correct Answer: A. Ulcerative pain

Explanation: Epigastric pain that is rhythmic, cramp-like, and occurs during the day without late postprandial irradiation is most suggestive of ulcerative pain. This type of pain is often associated with peptic ulcers, where the discomfort tends to follow a specific pattern and can be relieved by food or antacids. Other conditions like gastritis, gastroesophageal reflux, acute hemorrhagic pancreatitis, and hepatic colic have different pain characteristics or associated symptoms.

  1. Topic: Diarrhea Due to Malabsorption

Correct Answer: A. Significant weight loss

Explanation: Diarrhea due to malabsorption is often associated with significant weight loss because the body is unable to absorb essential nutrients effectively, leading to malnutrition. While steatorrhea (fatty stools) can also indicate malabsorption, significant weight loss is a more direct and prominent sign of the condition. Mucous and bloody diarrhea are more typical of inflammatory or infectious causes, and stool weight at 150g/d is generally not indicative of malabsorption, as it is more relevant to osmotic or secretory types of diarrhea.

  1. Topic: Isolated Functional Colonopathy

Correct Answer: B. Weight loss

Explanation: Isolated functional colonopathy, such as irritable bowel syndrome (IBS), typically presents with symptoms like abdominal pain, constipation, diarrhea, and postprandial bloating. Weight loss is not a common feature of functional colonopathy and often suggests other underlying issues or conditions.

  1. Topic: Acute Pancreatitis

Correct Answer: B. Cholelithiasis

Explanation: Cholelithiasis (gallstones) is one of the most common causes of acute pancreatitis. While alcohol is also a significant cause, gallstones are more frequently associated with acute episodes. Other factors such as obesity, clofibrate use, and hyperparathyroidism are less common causes.

  1. Topic: Nephrotic Syndrome

Correct Answer: C. An increase in serum gammaglobulins

Explanation: Nephrotic syndrome typically features hypoalbuminemia, hypercholesterolemia, a risk of thrombosis, and often a low urinary sodium concentration. However, an increase in serum gammaglobulins is not a typical feature of nephrotic syndrome; instead, it is often characterized by hypoglobulinemia due to protein loss in the urine.

  1. Topic: Nephritic Syndrome

Correct Answer: B. Hypo protidemia less than 60g per liter

Explanation: Nephritic syndrome is characterized by hypertension, hematuria with cylindruria, edema, and persistent proteinuria. However, hypo protidemia (specifically less than 60g/L) is more commonly associated with nephrotic syndrome rather than nephritic syndrome.

  1. Topic: Uric Acid Stone Dissolution

Correct Answer: B. An alkaline urinary pH

Explanation: Dissolving a uric acid stone requires an alkaline urinary pH. Uric acid stones form in acidic urine, so alkalinizing the urine helps to dissolve these stones by increasing their solubility.

  1. Topic: Acute Prostatitis

Correct Answer: D. A hard and stony prostatitis

Explanation: Acute prostatitis typically presents with painful prostatitis, acute urinary retention, urinary burns, and a high fever. A “hard and stony” prostate is more characteristic of chronic prostatitis or prostate cancer, not acute prostatitis.

  1. Topic: Pyuria (Pus in the Urine)

Correct Answer: C. Glomerulonephritis

Explanation: Pus in the urine (pyuria) is commonly associated with infections or conditions like renal lithiasis, vesico-renal reflux, a retained bladder, or cystitis. Glomerulonephritis, however, is primarily an inflammation of the glomeruli and typically presents with hematuria (blood in the urine) rather than pyuria.

  1. Topic: Contraindications for Biguanides in Diabetes Mellitus

Correct Answer: B. Kidney failure

Explanation: Biguanides, such as metformin, are contraindicated in patients with kidney failure due to the increased risk of lactic acidosis, a serious and potentially life-threatening condition. Obesity, insulin-dependent diabetes, angina, and peripheral neuropathy do not directly contraindicate the use of biguanides.

  1. Topic: Complications of Type 4 Hypertriglyceridemia

Correct Answer: B. Acute pancreatitis

Explanation: Type 4 hypertriglyceridemia, characterized by elevated triglyceride levels, can lead to acute pancreatitis, a serious and potentially life-threatening condition. While atheroma, hypertension, obesity, and gout are associated with lipid disorders, they are not specific complications of Type 4 hypertriglyceridemia like acute pancreatitis.

  1. Topic: Clinical Signs of Acromegaly

Correct Answer: D. Prognathism of the lower jaw

Explanation: The most suggestive clinical sign of acromegaly is prognathism of the lower jaw, which refers to the abnormal protrusion of the lower jaw. This feature is a hallmark of acromegaly due to excessive growth hormone, leading to characteristic facial changes. Swelling of the fingers, weight gain, hair loss, and exophthalmos may be present but are not as specific to acromegaly as prognathism.

  1. Topic: Biochemical Parameters Indicative of Conn’s Adenoma

Correct Answer: B. Hypokalemia

Explanation: Hypokalemia is a key biochemical parameter that suggests the presence of a Conn’s adenoma (primary hyperaldosteronism) in a hypertensive patient. Conn’s adenoma leads to excessive secretion of aldosterone, causing the kidneys to retain sodium (leading to hypernatremia) and excrete potassium, resulting in hypokalemia. The other options are less specific or unrelated to Conn’s adenoma.

  1. Topic: Erysipelas

Correct Answer: E. It has a gradual, non-painful onset

Explanation: Erysipelas typically presents with a sudden onset of symptoms, including pain, redness, and swelling in the affected area. The condition is often painful, and the rapid progression of symptoms contradicts the idea of a gradual, non-painful onset, making option E the incorrect statement among the provided options.

  1. Topic: Candida Infections

Correct Answer: C. Ringworm of the scalp

Explanation: Candida species are responsible for infections such as vaginitis, thrush, black hairy tongue, and intertrigo. However, ringworm of the scalp (tinea capitis) is caused by dermatophytes like Trichophyton or Microsporum, not by Candida. Thus, option C is the correct answer.

  1. Topic: Paget’s Disease

Correct Answer: E. Osteonecrosis

Explanation: Paget’s disease is a bone disorder that can lead to complications such as deafness, heart failure, headaches, and hip disease due to abnormal bone growth and deformation. However, osteonecrosis (bone death due to poor blood supply) is not a typical complication of Paget’s disease. Thus, option E is the correct answer.

  1. Topic: Acute Gout in Chronic Renal Failure

Correct Answer: C. Colchicine

Explanation: For an acute attack of gout, especially in patients with chronic renal failure, colchicine is often used as it effectively reduces inflammation during an acute attack. Aspirin and phenylbutazone are not recommended due to their potential to worsen kidney function or interact adversely with renal conditions. Benzobromarone and allopurinol are used for chronic management and prevention of gout but are not suitable for treating acute attacks. Thus, option C is the most appropriate choice.

  1. Topic: Diagnosis of Syphilitic Chancre

Correct Answer: E. Nelson’s test

Explanation: For diagnosing a syphilitic chancre, useful tests include ultramicroscopic examination (A) to visualize the spirochetes, the VDRL (B) for initial screening, TPHA (C) for confirmation of syphilis, and immunofluorescence (D) for detecting Treponema pallidum. Nelson’s test (E), which is not typically used for diagnosing syphilis, is therefore the least relevant in this context.

Top of Form

  1. Topic: Persistent Salmonella Carrier After Typhoid Fever

Correct Answer: B. Cholelithiasis

Explanation: Salmonella Typh 1 can persist in the gallbladder of patients, leading to chronic carriage. Therefore, cholelithiasis (B) is a common finding as the bacteria may reside in the gallbladder and contribute to the formation of gallstones. Splenic abscess (A), stomach ulcer (C), Meckel’s diverticulum (D), and appendicular abscess (E) are not typically associated with persistent Salmonella carriage after typhoid fever.

  1. Topic: Tetanus Prevention in a Vaccinated Individual

Correct Answer: D. A toxoid booster

Explanation: In individuals who are adequately vaccinated but who have not received a tetanus booster in the past 10 years, a single booster of the tetanus toxoid (D) is recommended. The initial vaccination at age 19 and the booster at 20 should provide long-term immunity, but routine boosters every 10 years are necessary to maintain protection. The human tetanus immunoglobulin (B, E) is only used in unvaccinated individuals or those with unknown vaccination status presenting with high-risk wounds. Two toxoid injections (C) are not required for someone with a recent vaccination history.

  1. Topic: Antibiotic for Gas Gangrene

Correct Answer: C. Penicillin G

Explanation: Gas gangrene, typically caused by Clostridium species, is effectively treated with Penicillin G (C), which targets these anaerobic bacteria. Gentamycin (A) and Cotrimoxazole (D) are not primarily effective against Clostridia. Colistin (B) is used for Gram-negative infections, and Nalidixic acid (E) is used for urinary tract infections, making them unsuitable for gas gangrene.

  1. Topic: Diagnosis of Cerebral Toxoplasmosis in HIV-Infected Patients

Correct Answer: D. Meningeal stiffness

Explanation:

Cerebral toxoplasmosis in HIV-infected patients is characterized by symptoms such as convulsive seizures (A), fever (B), hemiparesis (C), and altered consciousness (E). Meningeal stiffness (D), typically associated with meningitis, is not a common feature of cerebral toxoplasmosis. The presentation of this condition usually does not include meningeal signs but rather neurological deficits and systemic symptoms.

  1. Topic: Blood Findings in Myeloma (Kahler’s Disease)

Correct Answer: E. None of the above

Explanation:

In the typical form of myeloma, or Kahler’s disease, blood findings typically include abnormalities such as monoclonal gammopathy, increased levels of paraproteins (M proteins), and anemia, but not the specific options listed. Plasma cells may be found in the bone marrow (A), but they are not typically seen in blood in significant numbers. Eosinophilia (B), hyperlymphocytosis (C), and neutrophilic polynucleosis (D) are not characteristic of myeloma. Therefore, the answer is that none of these findings are typically observed in the blood of a patient with myeloma.

  1. Topic: Hemopathies and Splenomegaly

Correct Answer: B. Chronic myeloid leukemia

Explanation: Chronic myeloid leukemia (B) is commonly associated with splenomegaly due to the accumulation of abnormal leukocytes in the spleen. Biermer’s disease (A), also known as pernicious anemia, does not typically cause splenomegaly. Kahler’s disease (C) and hemophilia (D) are not characteristically associated with splenomegaly, and latrogenic agranulocytosis (E) involves a decreased white blood cell count but not necessarily splenomegaly.

  1. Topic: Thrombocytopenia in Hepatic Cirrhosis

Correct Answer: D. Hypersplenism

Explanation: In hepatic cirrhosis with signs of portal hypertension, thrombocytopenia (low platelet count) is commonly due to hypersplenism (D), where the spleen is enlarged and sequesters platelets, leading to a reduced number in circulation. Bone marrow failure (A) is less likely given the normal D-dimer levels and absence of signs of bleeding. Disseminated intravascular coagulation (DIC) (B) typically shows elevated D-dimer levels, not low. Immunological destruction (C) and consumption at the level of esophageal varices (E) are less common causes in this context.

PEDIATRICS

  1. Topic: Corticosteroid Therapy in Acute Rheumatic Fever (ARF)

Correct Answer: A. The decrease in SV

Explanation: In the management of acute rheumatic fever (ARF) with corticosteroids, the decision to reduce therapy is typically guided by clinical improvement. A decrease in the severity of symptoms (SV) such as inflammation or joint swelling supports tapering of corticosteroids. Increased leukocytosis (B) indicates ongoing inflammation, suggesting that corticosteroids might need to be continued or adjusted. Decreases in antistreptokinases (C) and antistreptolysins (D) indicate a reduction in streptococcal activity, but they are not as direct indicators for adjusting corticosteroid therapy. The throat swab result (E) is more relevant for diagnosing streptococcal infection rather than adjusting corticosteroid therapy.

  1. Topic: Risk of Osler Endocarditis in Congenital Heart Diseases

Correct Answer: C. Interauricular communication

Explanation: Osler endocarditis, also known as infective endocarditis, is a condition that can affect congenital heart diseases with turbulent blood flow or abnormal heart structures. Interauricular communication (C), or atrial septal defect, generally carries a lower risk for Osler endocarditis compared to other congenital heart defects. Tetralogy of Fallot (A), persistent ductus arteriosus (B), ventricular communication (D), and coarctation of the aorta (E) are associated with higher risks due to turbulent blood flow and increased susceptibility to infections in these areas.

  1. Topic: Generalized Cyanosis

Correct Answer: D. An excess of reduced hemoglobin

Explanation: Generalized cyanosis is characterized by a bluish discoloration of the skin and mucous membranes due to an excess of reduced (deoxygenated) hemoglobin in the blood (D). This condition typically arises when there is insufficient oxygenation of blood, which can occur with severe hypoxia (C) or conditions affecting oxygen transport. Alveolar hypoventilation (A) and hypercapnia (B) can contribute to hypoxia, but the direct cause of cyanosis is the excess of reduced hemoglobin. Polycythemia (E) can increase blood viscosity but does not directly cause cyanosis.

  1. Topic: Viral Bronchopneumopathy in Infants

Correct Answer: D. Acute respiratory failure

Explanation: In infants with viral bronchopneumopathy, acute respiratory failure (D) is the primary concern due to their smaller airways and less efficient respiratory systems, which can quickly become overwhelmed by the infection. While bacterial superinfection (A) and heart failure (B) are also potential complications, the immediate risk is the inability to maintain adequate oxygenation and ventilation. Pneumothorax (C) and interstitial emphysema (E) are less common but can occur in severe cases.

  1. Topic: Primary Tuberculosis Infection in Children

Correct Answer: A. An erythema nodosum

Explanation:

In children, primary tuberculosis infection can present with erythema nodosum (A), a skin condition often associated with tuberculosis. This finding is more specific to tuberculosis compared to the other options. Diarrhea (B), angina with hyperlymphocytosis (C), and wheezing dyspnea (D) are not typically associated with primary tuberculosis in children. While they may occur in various infections or conditions, erythema nodosum is a more indicative sign of tuberculosis.

  1. Topic: BCG Vaccine

Correct Answer: D. A live but attenuated strain of Mycobacterium bovis

Explanation: The BCG (Bacillus Calmette-Guérin) vaccine is composed of a live but attenuated strain of Mycobacterium bovis (D). This strain is weakened so it does not cause disease but stimulates an immune response. It is not a killed strain (C) or an extract of heated bacilli (B), and it does not involve a mixture of different mycobacteria (E). The vaccine’s live attenuated nature is crucial for its effectiveness in providing immunity against tuberculosis.

  1. Topic: Asthma Onset in Children Before Age 5

Correct Answer: E. He is very often corticodependent

Explanation: Asthma in children before age 5 is often of allergic origin (A) and commonly associated with other atopic conditions such as eczema or allergic rhinitis (C). It can sometimes resolve or improve significantly by puberty (B), and it can lead to chest deformities if left untreated over a long period (D). However, it is not necessarily “very often corticodependent” (E); many children with early-onset asthma may not require long-term corticosteroid therapy, especially if their symptoms are well-managed or if they experience significant improvement with age.

  1. Topic: Treatment of Gastroesophageal Reflux Disease in Infants

Correct Answer: A. Administration of anticholinergics

Explanation: In managing gastroesophageal reflux disease (GERD) in infants, the following strategies are useful: administration of metoclopramide (B), which helps in gastric emptying; positioning the infant in a prone position or in dorsal decubitus with a 45-degree incline (C and D), which can reduce reflux; and thickening milk to reduce regurgitation (E). However, anticholinergics (A) are not typically used in the treatment of GERD in infants due to their side effect profile and lack of efficacy in this context.

  1. Topic: Hematemesis in Infants

Correct Answer: D. Peptic esophagitis

Explanation: In infants, the most common cause of hematemesis is peptic esophagitis (D), which can result from acid reflux or irritation of the esophagus. Conditions like gastric ulcers (A) and Meckel’s diverticulum (E) are less common causes. Cow’s milk protein intolerance (B) and thrombocytopenia (C) can cause gastrointestinal symptoms but are not the primary causes of hematemesis in infants. Peptic esophagitis stands out as the most frequent culprit in this context.

  1. Topic: Dysenteric Syndrome in Infants

Correct Answer: C. Salmonella

Explanation: Dysenteric syndrome in infant stools, characterized by frequent and painful diarrhea with blood and mucus, is most commonly associated with infections caused by Salmonella (C). Rotavirus (A) typically causes watery diarrhea rather than dysenteric syndrome. Cervical bacillus (B) and Candida albicans (D) are less commonly linked to dysenteric diarrhea in infants. Therefore, Salmonella is the primary pathogen associated with this condition in infants.

  1. Topic: Recommendations for Newborns of HBsAg and HBeAg Positive Mothers

Correct Answer: E. Serovaccination

Explanation: For a newborn whose mother is positive for both HBsAg and HBeAg, the recommended approach is serovaccination (E). This involves administering both hepatitis B vaccine and hepatitis B immunoglobulin (HBIG) to the newborn. This combined strategy provides immediate passive immunity and active vaccination to protect against hepatitis B virus infection. Options such as vaccination only (C) or specific immunoglobulins alone (D) are insufficient for full protection. Polyvalent immunoglobins (B) and no treatment (A) are not appropriate for this high-risk situation.

  1. Topic: Diagnosis of Urinary Tract Infection in Children

Correct Answer: D. Uroculture 1,000,000 germs/ml 5,000 leukocytes/ml

Explanation: The diagnosis of a urinary tract infection (UTI) in children is supported by a high bacterial count and a significant presence of leukocytes. Option D indicates a uroculture with 1,000,000 germs/ml and 5,000 leukocytes/ml, which is indicative of a UTI due to the high bacterial count and elevated leukocytes, reflecting an infection and inflammation. Lower bacterial counts (A, B, C) with or without some leukocytes are not typically sufficient for diagnosing a UTI. Option E is incorrect because not all listed results are diagnostic for a UTI.

  1. Topic: Diagnosis of Recurrent Pyelonephritis with Urography Findings

Correct Answer: B. Retrograde and voiding cystography

Explanation: In a case of recurrent pyelonephritis with no visible obstruction on urography but with renal atrophy and abnormal calyces, retrograde and voiding cystography are essential for diagnosing potential vesicoureteral reflux (VUR). VUR is a condition where urine flows backward from the bladder into the ureters and kidneys, which can lead to recurrent infections and renal damage. This test helps identify reflux, which is crucial for managing and preventing further renal damage. Other options like kidney biopsy (A), arteriography (C), lymphography (D), and ultrasound (E) are less specific in diagnosing VUR, which is a common underlying issue in such cases.

  1. Topic: Diagnosis of Hematuria in Children

Correct Answer: D. Valves of the posterior urethra

Explanation: Hematuria in children can be associated with conditions like glomerulonephritis (A), Berger’s disease (B), and urinary lithiasis (C). These conditions commonly cause blood in the urine due to their effects on the renal and urinary systems. However, valves of the posterior urethra (D) primarily cause obstructive symptoms and may lead to urinary tract infections or hydronephrosis rather than hematuria directly. Hence, the discovery of hematuria is less commonly linked to this condition.

  1. Topic: Childhood Nephrotic Syndrome

Correct Answer: A. It most often cortico-sensitive

Explanation: Childhood nephrotic syndrome is typically responsive to corticosteroid therapy (A), which is a key aspect of its management. Most cases of nephrotic syndrome in children do not require a renal biopsy before starting corticosteroid therapy (C) and are unlikely to progress to chronic renal failure (D). Although relapses can occur, the condition often does not require a sodium-restricted diet outside of flare-ups (E).

  1. Topic: Impetigo

Correct Answer: E. It requires a bacteriological sample

Explanation: Impetigo is a highly contagious skin infection commonly affecting children (A) and typically involves crusty lesions around the face and mouth (C), and can sometimes present with bullous forms (D). It is generally advised to exclude affected children from school to prevent spread (B). While bacteriological samples can be helpful for diagnosis, they are not always required for clinical management (E). The diagnosis is often based on clinical presentation.

  1. Topic: Infantile Scabies

Correct Answer: C. Involvement of the face

Explanation: Infantile scabies is characterized by pruritus (B) and lesions in the interdigital spaces (A) and on the soles of the feet (D). It is known for its widespread involvement in areas where adults might not typically see scabies, such as the soles and palms. However, it usually does not involve the face (C), which helps differentiate it from other skin conditions affecting infants. Thus, the face is not typically affected in infantile scabies, making C the correct answer.

  1. Topic: Atopic Eczema in Infants

Correct Answer: A. It begins in the first month of life

Explanation:

Atopic eczema in infants typically does not begin within the first month of life (A). Instead, it often starts after the first few months of life and commonly begins on the face (C). The condition evolves with periods of flare-ups and remissions (B) and presents with erythemato-vesicular lesions (D). A family history of allergy is frequently observed (E), which aligns with the genetic component of atopic conditions. Thus, A is the exception as it is not characteristic of atopic eczema’s typical onset.

  1. Topic: Tetanus Prophylaxis

Correct Answer: E. None of the above

Explanation: For a 7-year-old child who has been vaccinated correctly and has a minor scratch, routine prophylaxis typically involves checking the child’s tetanus vaccination status. If the child is up-to-date with vaccinations and the wound is minor, no additional prophylaxis is generally needed (E). Specific prophylaxis options like seroprophylaxis (B) or additional vaccination (A) are usually reserved for cases with higher risk or incomplete vaccination history. The correct choice is E because none of the listed options are necessary for a properly vaccinated child with a minor injury.

  1. Topic: Measles Clinical Manifestations

Correct Answer: A. Dissociation of pulse and temperature

Explanation: Measles typically presents with a constellation of symptoms including conjunctivitis with photophobia (B), fever that often persists beyond the rash onset (C), redness and petechiae of the soft palate (D), and symptoms like coryza and cough (E). Dissociation of pulse and temperature (A) is not characteristic of measles; in fact, fever and pulse are usually correlated. Therefore, A is the correct answer as it does not align with the typical clinical manifestations of measles.

  1. Topic: Kolick Sign

Correct Answer: C. Seeding of whitish spots on the oral mucosa

Explanation: The Kolick sign refers to the appearance of whitish spots on the oral mucosa, which are indicative of certain viral infections like measles. Options A, B, D, and E describe other clinical signs that do not correspond to the Kolick sign. Specifically, ulceration of the tonsils (A), gingival ulcers (B), purpura (D), and vesicles (E) are associated with different conditions or symptoms and do not define the Kolick sign.

  1. Topic: Mumps Invasion Phase

Correct Answer: C. Turgor of the orifice of Stenon’s canal

Explanation: During the mumps invasion phase, the presence of turgor (swelling or tenderness) at the orifice of Stenon’s canal, which is the duct of the parotid gland, is a key diagnostic sign. This is due to inflammation of the parotid glands, leading to obstruction or swelling around this area. The other options—high fever with headache (A), erythematous pharyngitis (B), pre-tragal lymphadenopathy (D), and hypersalivation (E)—are less specific and can be associated with various conditions, not just mumps.

  1. Topic: Facial Rash in a 9-Month-Old Child

Correct Answer: C. Herpes virus

Explanation: The facial rash resembling Kaposi-Juliusberg varioliform pustulosis in a 9-month-old child is caused by the herpes virus, particularly HSV (Herpes Simplex Virus). This condition can present with vesicular or pustular lesions on the skin. Varicella zoster virus (A) causes chickenpox, measles virus (B) leads to measles, and Staphylococcus (D) can cause impetigo but not the described rash. The term “unknown” (E) is not applicable as the herpes virus is a well-recognized cause.

  1. Topic: Congenital Toxoplasmosis

Correct Answer: A. It is more frequent during the 3rd trimester of pregnancy

Explanation: Congenital toxoplasmosis occurs when the Toxoplasma gondii infection is transmitted from the mother to the fetus. The risk of transmission increases as the pregnancy progresses, making it more frequent in the 3rd trimester (A). While it is less common in the 1st trimester (B), the severity is not necessarily greater if contracted later in pregnancy; instead, earlier infections are often more severe. Pyrimethamine is used for treatment, but this is relevant for treating the infection, not the frequency of transmission (D). Fibroplasia is not the characteristic eye lesion in newborns; chorioretinitis is more typical (E).

  1. Topic: Acute Leukemia in Children

Correct Answer: B. Lymphoblastic

Explanation: Acute lymphoblastic leukemia (ALL) is the most common type of acute leukemia in children, characterized by the proliferation of immature lymphoid cells (B). Acute myeloblastic leukemia (A) and other types such as monoblastic (C), promyelocytic (D), and erythroleukemia (E) are less common in the pediatric population.

  1. Topic: Purpura in Children

Correct Answer: E. Rheumatoid purpura

Explanation: Rheumatoid purpura, also known as Henoch-Schönlein purpura, is characterized by purpura on the lower limbs, often accompanied by fever and a generally good condition (E). Acute leukemia (A) and Waldenstrom macroglobulinemia (B) are less common causes of purpura in this context. Flea bites (C) and monocyte angina (D) are unlikely to present with such systemic symptoms and purpura.

  1. Topic: Hemophilia A

Correct Answer: D. Active partial thromboplastin time (APTT)

Explanation: In untreated hemophilia A, the Active Partial Thromboplastin Time (APTT) is prolonged due to deficiencies in clotting factors involved in the intrinsic pathway of hemostasis (D). Bleeding time (A), platelet count (B), Quick time (C), and thrombin time (E) are usually normal in hemophilia A. The APTT test is specifically sensitive to deficiencies in factor VIII, which is characteristic of hemophilia A.

  1. Topic: Routine Vaccinations in Children

Correct Answer: C. Mumps

Explanation: In routine childhood vaccinations, children are vaccinated against pertussis (A), pneumococcal infections (B), Haemophilus influenzae type b infection (D), and diphtheria (E). Mumps (C) is included in the MMR (measles, mumps, rubella) vaccine, but it is not a standalone vaccination and is not separately emphasized in routine schedules as frequently as the others.

  1. Topic: Medical Outcomes in Drowning

Correct Answer: C. Hypoxic brain damage

Explanation: In drowning, hypoxic brain damage (C) is the most serious medical outcome due to prolonged oxygen deprivation, leading to potential severe and irreversible neurological damage. Pulmonary edema (A) and acute renal failure (B) can occur, but they are secondary concerns compared to the immediate and profound impact of hypoxia on the brain. Blood loss (D) and cardiac arrhythmias (E) are less directly related to the primary consequences of drowning.

  1. Topic: Causes of Sensory Hearing Loss in Children

Correct Answer: D. Acute otitis media

Explanation: Sensory hearing loss is primarily linked to conditions affecting the inner ear or auditory nerve. Acute otitis media (D) typically causes conductive hearing loss rather than sensory loss, affecting the middle ear. In contrast, rubella (A), meningitis (B), perinatal asphyxia (C), and the use of aminoglycosides (E) can lead to sensory hearing loss by impacting the inner ear or auditory pathways.

  1. Topic: Complications of Perinatal Asphyxia

Correct Answer: B. Hyperemesis gravidarum

Explanation: Perinatal asphyxia is associated with complications that directly impact the fetus and the delivery process. Abrupt placenta (A), prematurity (C), pre-eclampsia (D), and amniotic fluid tinged with meconium (E) are complications related to or indicative of issues that can lead to perinatal asphyxia. Hyperemesis gravidarum (B), however, is a severe form of nausea and vomiting during pregnancy that does not directly relate to perinatal asphyxia.

  1. Topic: Marfan Syndrome

Correct Answer: A. Aortic dilatation and loose joints suggestive of connective tissue disorder

Explanation: Marfan syndrome is a genetic disorder affecting connective tissue, which often presents with aortic dilatation and joint laxity (loose joints) (A). The other options relate to different conditions: excessive bone length and width suggest bone metabolism disorders (B), tendons and ligaments issues point to muscle metabolism disorders (C), and large size with excess subcutaneous tissue is indicative of growth factor disorders (D). Marfan syndrome is primarily characterized by connective tissue abnormalities, making option A the correct choice.

  1. Topic: Melena in an 18-Year-Old

Correct Answer: E. Meckel’s diverticulum

Explanation: Meckel’s diverticulum is a congenital abnormality that can cause gastrointestinal bleeding and melena (black, tarry stools) due to the presence of ectopic gastric mucosa or ulceration. This condition can present with anemia and shock due to significant blood loss. Eosinophilic colitis (A) and hemolytic-uremic syndrome (B) are less likely to cause melena. Ulcerative colitis (C) usually presents with diarrhea and rectal bleeding, and juvenile polyps (D) are less commonly associated with such acute presentations. Hence, Meckel’s diverticulum is the most likely diagnosis in this case.

  1. Topic: Prolonged Bleeding Time

Correct Answer: A. Hemophilia A

Explanation: Hemophilia A is a coagulation disorder characterized by a deficiency in clotting factor VIII, which primarily affects the clotting cascade and not the initial platelet function, thus it does not typically cause prolonged bleeding time. In contrast, Von Willebrand disease (B) affects both platelet function and clotting, resulting in prolonged bleeding time. Aspirin-induced thrombocytopathy (C) impairs platelet aggregation, extending bleeding time. Bernard-Soulier syndrome (D) involves platelet dysfunction, leading to prolonged bleeding time. Idiopathic thrombocytopenic purpura (E) results in reduced platelet count but can also affect bleeding time, making Hemophilia A the exception here.

  1. Topic: Diabetic Ketoacidosis (DKA) in Type 1 Diabetes

Correct Answer: A. Overfeeding

Explanation: Diabetic ketoacidosis (DKA) is primarily triggered by factors that increase insulin requirements or decrease insulin effectiveness. Vomiting (B) can lead to dehydration and electrolyte imbalances, which can exacerbate DKA. Omission of insulin doses (C) directly leads to uncontrolled hyperglycemia and ketogenesis. Infection (D) can increase insulin needs and precipitate DKA due to inflammatory stress. Lack of patient education (E) can result in poor management and missed insulin doses. Overfeeding (A) is less likely to directly cause DKA compared to these other factors, making it the exception.

  1. Topic: Delayed Skeletal Maturation in Adolescents

Correct Answer: B. Psychosocial deprivation

Explanation: Delayed skeletal maturation in adolescents can be due to several underlying conditions. Growth hormone deficiency (A) affects bone growth and development, leading to delayed maturation. Hypothyroidism (C) can result in delayed skeletal development due to its impact on metabolic processes. Late (non-classic) onset of 21-hydroxylase deficiency (D) affects adrenal steroid production, which can impact bone age. Constitutional delay of puberty (E) is a common cause of delayed skeletal maturation, as it affects the timing of puberty and skeletal development. Psychosocial deprivation (B) is less directly related to bone age delays compared to the other conditions, making it the exception.

  1. Topic: Newborn Hypothyroidism Screening

Correct Answer: E. None of the above

Explanation: Newborn hypothyroidism screening aims to ensure normal linear growth (A), support normal intellectual function (B), and can aid in genetic counseling (C). Although screening does not directly prevent sudden infant death syndrome (D), it is still crucial in preventing long-term complications by addressing hypothyroidism early. Therefore, “None of the above” (E) is correct because all the listed goals are relevant to newborn hypothyroidism screening.

  1. Topic: Pheochromocytoma in Children

Correct Answer: E. Sufficiency

Explanation: Pheochromocytoma, a tumor of the adrenal glands, can cause symptoms like headache (A), weight loss (B), convulsions (C), and sweating (D) due to excess catecholamines. However, “sufficiency” (E) is not a recognized symptom of pheochromocytoma. The condition is associated with excess hormone secretion, not sufficiency.

  1. Topic: Rett Syndrome

Correct Answer: D. Autosomal recessive inheritance

Explanation: Rett syndrome is a neurodevelopmental disorder characterized by autistic behavior (A), microcephaly (B), hand-wringing movements (C), and progressive dementia (E). However, Rett syndrome is typically inherited in an X-linked dominant pattern, not autosomal recessive (D). The inheritance pattern distinguishes Rett syndrome from other genetic disorders.

  1. Topic: Spina Bifida Cystica

Correct Answer: B. It is fatal if not treated within 24 hours

Explanation: Spina bifida cystica is a type of neural tube defect often associated with hydrocephalus (A), which can be diagnosed in utero by ultrasound (C). It can lead to urological problems (D) and often requires orthopedic care (E). However, while prompt treatment is important, it is not necessarily fatal if not treated within 24 hours; rather, early intervention helps reduce the risk of severe complications. Therefore, statement B is incorrect.

PUBLIC HEALTH

  1. Topic: Study Design

Correct Answer: C. A prospective cohort

Explanation: In a prospective cohort study, participants are followed over time to observe outcomes based on exposure status. In this case, women with a history of smoking are followed to determine their babies’ birth weights. This design allows for tracking the association between smoking and birth weight over time. Other designs like clinical trials (A), cross-sectional studies (B), case-control studies (D), and retrospective cohort studies (E) do not fit this description as closely as the prospective cohort design.

  1. Topic: Blood Pressure Measurement and Study Design

Correct Answer: B. Regression to the mean

Explanation: Regression to the mean occurs when subjects with extreme measurements at the start of a study tend to have more average measurements upon re-evaluation, simply due to statistical variability. In this case, patients with high diastolic blood pressure may show a reduction over time, leading to fewer patients meeting the study’s criteria upon follow-up. Spontaneous resolution (A) and measurement error (D) are less likely to fully explain the reduction in qualifying patients, while basic drift (C) and the Hawthorne effect (E) do not directly address the statistical phenomenon observed.

  1. Topic: Incidence Calculation

Correct Answer: D. Person-years of observation

Explanation: Person-years of observation is used as the denominator in calculating incidence rates to account for both the number of people and the time each person is at risk. This measure helps to reflect the time each individual contributes to the risk of developing the disease. The number of new cases observed (B) is the numerator for incidence calculation, while the number of cases observed (A) and asymptomatic cases (C) are not directly used. Persons lost to follow-up (E) do not contribute to the denominator but can affect the study’s accuracy.

  1. Topic: Normal Distribution and Weight Analysis

Correct Answer: D. 16%

Explanation: To determine the percentage of women who weigh more than 59.5 kg, we need to find the z-score for 59.5 kg using the mean (52 kg) and standard deviation (7.5 kg).

The z-score is calculated as:

z=(X−mean)standard deviation=(59.5−52)7.5=1z = \frac{(X – \text{mean})}{\text{standard deviation}} = \frac{(59.5 – 52)}{7.5} = 1z=standard deviation(X−mean)​=7.5(59.5−52)​=1

A z-score of 1 corresponds to approximately the 84th percentile of the normal distribution, meaning that about 16% of women weigh more than 59.5 kg.

 

 

  1. Topic: Measures of Central Tendency

Correct Answer: E. 3, 4, 3

Explanation: To find the mean, median, and mode:

  1. Mean: Sum of all values divided by the number of values. Mean=4+6+2+2+4+3+2+1+79=309=3.33\text{Mean} = \frac{4 + 6 + 2 + 2 + 4 + 3 + 2 + 1 + 7}{9} = \frac{30}{9} = 3.33Mean=94+6+2+2+4+3+2+1+7​=930​=3.33 Approximated to the nearest integer, the mean is 3.
  2. Median: The middle value when the numbers are sorted in ascending order. Sorted numbers: 1, 2, 2, 3, 4, 4, 6, 7 The median is 4.
  3. Mode: The value that appears most frequently. In this case, the mode is 3 (as 3 appears most frequently).

Thus, the mean is 3, the median is 4, and the mode is 3.

  1. Topic: Analyzing Data for Association Studies

Correct Answer: C. Fisher’s exact test

Explanation: Fisher’s exact test is used to determine if there are nonrandom associations between two categorical variables, especially in cases where sample sizes are small. In this scenario, where the association between coffee consumption (more than eight cups vs. not drinking coffee) and hypertension is being assessed, Fisher’s exact test is appropriate for analyzing the categorical data from a 2×2 contingency table. The Chi-square test (A) is typically used for larger sample sizes. The McNemar test (B) is used for paired nominal data, which doesn’t apply here. Student’s t-test (D) is used for comparing means between two groups, and Analysis of Variance (E) is used for comparing means among more than two groups.

  1. Topic: Incidence Rate Calculation

Correct Answer: D. 10

Explanation: To calculate the incidence rate per 1000 people per year in households with a culture-positive case at the first survey, follow these steps:

  1. Determine the number of new cases in the households with a culture-positive case during the follow-up period.
  2. Find the total number of people at risk in those households.
  3. Calculate the incidence rate using the formula: Incidence rate=Number of new casesTotal person-years at risk×1000\text{Incidence rate} = \frac{\text{Number of new cases}}{\text{Total person-years at risk}} \times 1000Incidence rate=Total person-years at riskNumber of new cases​×1000

If the study data results in 10 new cases per 1000 people per year, this is the incidence rate for the households with a culture-positive case from the first survey.

The other options (A, B, C, E) do not correctly reflect the given incidence rate calculation.

  1. Topic: Incidence Rate Calculation in Households Without Culture-Positive Cases

Correct Answer: C. 0.5

Explanation: To calculate the incidence rate of new cases per 1000 person-years in households that had no culture-positive cases at the first survey:

  1. Identify the number of new cases in these households during the follow-up period.
  2. Determine the total person-years at risk for these households.
  3. Apply the formula: Incidence rate=Number of new casesTotal person-years at risk×1000\text{Incidence rate} = \frac{\text{Number of new cases}}{\text{Total person-years at risk}} \times 1000Incidence rate=Total person-years at riskNumber of new cases​×1000

If the incidence rate is calculated to be 0.5 per 1000 person-years, this reflects the rate of new cases in households without initial culture-positive cases.

Other options (A, B, D, E) do not accurately represent the incidence rate of 0.5 per 1000 person-years.

  1. Topic: Relative Risk of Contracting TB

Correct Answer: E. 20

Explanation: To calculate the relative risk (RR) of contracting TB in households with culture-positive cases compared to those without:

  1. Determine the incidence rate of TB in households with culture-positive cases.
  2. Determine the incidence rate of TB in households without culture-positive cases.
  3. Apply the formula: Relative Risk (RR)=Incidence rate in exposed groupIncidence rate in unexposed group\text{Relative Risk (RR)} = \frac{\text{Incidence rate in exposed group}}{\text{Incidence rate in unexposed group}}Relative Risk (RR)=Incidence rate in unexposed groupIncidence rate in exposed group​

Given that the relative risk is 20, this indicates that individuals in households with culture-positive cases are 20 times more likely to contract TB compared to those in households without initial culture-positive cases.

Other options (A, B, C, D) do not accurately reflect the significant increase in risk associated with culture-positive households.

  1. Topic: Experimental vs. Observational Studies

Correct Answer: E. Investigators determine who will or will not be exposed to the suspected causative factor

Explanation: In an experimental study, such as a randomized controlled trial, the researchers actively intervene and assign participants to either the exposed or non-exposed group to study the effect of an intervention or exposure. This control over exposure is what distinguishes experimental studies from observational studies. In observational studies, researchers simply observe and record exposures and outcomes without assigning exposure.

  1. Topic: Rabies Prophylaxis in Children

Correct Answer: C. Administer rabies vaccine and rabies immunoglobulin

Explanation: Given the history of potential exposure to a bat, which is a known rabies vector, and the absence of visible scratches, it is still prudent to administer both rabies vaccine and rabies immunoglobulin. Rabies is a serious disease, and prophylactic treatment is essential even if no physical evidence of a bite or scratch is present. The combination of vaccine and immunoglobulin provides comprehensive protection against rabies.

  1. Topic: Vaccination During Pregnancy

Correct Answer: B. Varicella vaccine

Explanation: The varicella (chickenpox) vaccine is a live attenuated vaccine and is contraindicated during pregnancy due to the potential risk to the fetus. Pregnant women should avoid this vaccine, as live vaccines can pose a risk to the developing baby.

  1. Topic: Tetanus Prophylaxis

Correct Answer: A. No additional prophylaxis

Explanation: Given that the patient received the last dose of tetanus toxoid 7 years ago and has completed a primary series of immunizations, no additional prophylaxis is needed for this minor injury. Tetanus prophylaxis recommendations generally suggest a booster every 10 years, but since the injury is not classified as high risk and the patient is up-to-date on vaccinations, additional prophylaxis is not required.

  1. Topic: Interval Between Whole Blood Administration and MMR Vaccine

Correct Answer: D. 6

Explanation: It is recommended to wait at least 6 months after receiving whole blood before administering the measles-mumps-rubella (MMR) vaccine. This is because whole blood transfusions can interfere with the effectiveness of live vaccines like MMR. Waiting 6 months ensures that the vaccine will be effective.

  1. Topic: Etiological Agent of Lower Abdominal Pain and Cervical Findings

Correct Answer: B. Chlamydia trachomatis

Explanation: Chlamydia trachomatis is a common cause of cervicitis and can present with symptoms such as lower abdominal pain, friable cervix, and pain on cervical movement. This sexually transmitted infection often causes these findings without fever and can be associated with negative pregnancy tests.

  1. Topic: Risk of Infertility from Pelvic Inflammatory Disease (PID)

Correct Answer: D. 20%

Explanation: Pelvic inflammatory disease (PID) can increase the risk of infertility due to scarring and damage to the reproductive organs. Each episode of PID contributes to this risk, and after two episodes, the cumulative risk of infertility is around 20%. This risk reflects the potential for complications such as blocked fallopian tubes and other reproductive tract issues.

  1. Topic: Diagnosis of Genital Ulcers

Correct Answer: A. Syphilis

Explanation: A single, hard, painless ulcer on the penis that has been present for 2 days and is consistent with a primary chancre is characteristic of primary syphilis. The RPR test may be negative in the early stages of syphilis, as it can take several weeks for the antibodies to become detectable.

  1. Topic: Sexual Partner Notification

Correct Answer: B. Partners less than 30 days old

Explanation: For sexually transmitted infections (STIs) like syphilis, partners from the last 30 days should be informed and evaluated. This timeframe is based on the typical incubation period of STIs, ensuring that recent exposures are addressed effectively to prevent further transmission and complications.

  1. Topic: MMR Vaccination Safety

Correct Answer: A. A 15-month-old HIV infected child with a CD4 count of 700

Explanation: MMR (measles, mumps, rubella) vaccine is live attenuated and can be safely administered to children with HIV if their CD4 count is above 500 cells/mm³, indicating good immune function. A CD4 count of 700 suggests adequate immune status for receiving live vaccines.

  1. Topic: HCV-Positive Women Counseling

Correct Answer: B. The probability of transmission to the newborn is 5%

Explanation: Hepatitis C virus (HCV) transmission from mother to infant occurs in about 5% of cases, which is a key counseling point for HCV-positive women. This transmission rate helps in understanding the risk and planning appropriate follow-up for the newborn.

 

SURGERY

  1. Topic: Traumatic Posterior Shoulder Dislocation

Correct Answer: B. Attitude of arm in internal rotation

Explanation: In a traumatic posterior shoulder dislocation, the affected arm typically assumes an internal rotation position. This is because the posterior dislocation causes the humeral head to move backward, leading to the arm being held close to the body with internal rotation.

  1. Topic: Dislocation Erecta of the Shoulder

Correct Answer: B. Lower

Explanation: Dislocation erecta of the shoulder refers to a rare type of dislocation where the humeral head is displaced inferiorly or downward, causing the arm to be held in an extended position. This results in an unusual presentation where the arm is held in a position that appears as if it is standing upright, hence the term “erecta.”

  1. Topic: Carpal Tunnel Syndrome

Correct Answer: C. A paralysis of the opposition of the thumb

Explanation: Carpal tunnel syndrome affects the median nerve, which controls the muscles responsible for the opposition of the thumb. This results in difficulty performing tasks that require thumb opposition, such as pinching or grasping.

  1. Topic: Acute Osteomyelitis in Children

Correct Answer: D. Acute osteomyelitis in children is generally localized to the diaphyseal region of the bone

Explanation: Acute osteomyelitis in children typically affects the metaphyseal region of long bones, especially near the knee, rather than the diaphyseal region. This is because the metaphysis has a rich blood supply, making it more susceptible to hematogenous infection.

  1. Topic: Open Fracture Classification

Correct Answer: C. The loss of substance of the soft parts prevents any closure

Explanation: In the context of open fractures, Stage III is characterized by severe soft tissue injury where the loss of substance prevents any potential for closure of the wound. This indicates a more severe injury with extensive tissue damage.

  1. Topic: Synovial Fluid in Acute Gout

Correct Answer: D. Cloudy liquid with 10,000 cells per mm³ and very birefringent microcrystals

Explanation: During an acute gout attack, synovial fluid is typically cloudy and has a high cell count, often around 10,000 cells/mm³. The presence of very birefringent (needle-shaped) microcrystals, which are characteristic of monosodium urate crystals, confirms gout.

  1. Topic: Total Nerve Section

Correct Answer: B. Neurotmesis

Explanation: Neurotmesis refers to a complete transection or total section of a nerve, where both the nerve fibers and the surrounding connective tissue are disrupted. This results in a total loss of function and requires surgical intervention for possible repair.

  1. Topic: Posterior Shoulder Dislocation

Correct Answer: B. Impossibility of external rotation of the arm

Explanation: In posterior shoulder dislocation, the most suggestive clinical sign is the inability to externally rotate the arm. This occurs because the shoulder joint’s normal range of motion is restricted by the dislocated position.

  1. Topic: Femoral Diaphysis Fracture

Correct Answer: B. BA ECB

Explanation: A fracture of the femoral diaphysis commonly causes external rotation of the lower limb (B) due to muscle contractions. It may also lead to hemorrhagic shock (C) due to substantial bleeding and can be complicated by fat embolism (E). However, it does not typically present with an arch deformity of the thigh (A) or sciatic paralysis (D) before treatment. Sciatic paralysis is less common and usually occurs with more severe injury or complications.

  1. Topic: Isolated Functional Colopathy

Correct Answer: B. Weight loss

Explanation: Isolated functional colopathy, or functional gastrointestinal disorders, commonly present with symptoms like abdominal pain (A), constipation (C), diarrhea (D), and postprandial bloating (E). Weight loss (B) is not typically associated with functional colopathy and suggests a more serious underlying condition that requires further investigation.

  1. Topic: Evolution of a Subphrenic Abscess from Duodenal Ulcer Perforation

Correct Answer: B. BA BCD

Explanation: During the evolution of a subphrenic abscess resulting from duodenal ulcer perforation, you may observe: Fever (A) Subocclusive state (B) Vomiting (C) Localized defense (D) Pain on digital rectal examination (E) is not typically associated with subphrenic abscesses, as the abscess is generally not palpable or associated with rectal symptoms.

  1. Topic: Major Ischemic Risk Associated with Fracture

Correct Answer: C. AC BDE

Explanation: Rupture of the following arteries associated with a fracture focus creates a significant ischemic risk: The external iliac artery (A) The common femoral artery (C) The superficial femoral artery (D) The popliteal artery (E). The lower axillary artery (B) is not typically associated with major ischemic risk in the context of fractures. Thus, the correct combination is AC BDE.

  1. Topic: Septic Arthritis

Correct Answer: A. Osteophytes

Explanation: Septic arthritis typically presents with signs like articular pinching (B), mirror geodes (C), subchondral demineralization (D), and thickening of the soft tissues (E). These findings are associated with joint infection and inflammation. Osteophytes (A), however, are associated with osteoarthritis rather than septic arthritis. Hence, osteophytes are not a feature of septic arthritis, making this the correct answer.

  1. Topic: Prioritization of Surgical Interventions in Trauma

Correct Answer: C. Extra dural hematoma

Explanation: An extra dural hematoma (C) is a life-threatening condition that can cause rapid deterioration due to increased intracranial pressure. Immediate surgical intervention is crucial to prevent brain damage or death. While other injuries such as stomach wounds (A), open leg fractures (B), rupture of the urethra (D), and fractures of the acetabulum (E) are serious, they do not typically pose an immediate threat to life like an extra dural hematoma does. Therefore, the extra dural hematoma should be prioritized for surgical intervention.

  1. Topic: Fractures of the Upper Extremity of the Radius in Children

Correct Answer: D. They can accompany a dislocation of the elbow

Explanation: Fractures of the upper extremity of the radius in children, such as those involving the radial head, can indeed be associated with dislocations of the elbow (D). This combination often results from significant trauma to the elbow. While these fractures are common (A) and may involve the radial head or neck (B), and could potentially be associated with radial nerve injuries (C), they do not necessarily require surgical treatment (E) unless there are complications or significant displacement. Therefore, the ability to accompany an elbow dislocation is a key characteristic.

  1. Topic: Ankylosis

Correct Answer: B. Bone fusion between two articular surfaces

Explanation: Ankylosis specifically refers to the fusion of bone between two articular surfaces, resulting in immobility at the joint (B). This is different from morning stiffness (A) or decreased range of motion (C), which are symptoms rather than definitions. While ankylosis leads to a complete loss of movement at the joint (D), its defining feature is the actual fusion of the bones. Stiffness at the end of the day (E) is not related to the definition of ankylosis.

  1. Topic: Crural Nerve Paralysis

Correct Answer: B. The crural nerve passes under the crural arch medial to the femoral vein

Explanation: The crural nerve (femoral nerve) does not pass under the crural arch medial to the femoral vein; it actually passes lateral to the femoral vein (B). The nerve originates from the lumbar roots L2, L3, and L4 (A), divides into branches within the femoral triangle (C), and is involved in the patellar reflex arc (D). Hematoma of the psoas muscle can indeed affect the crural nerve (E).

  1. Topic: Lumbosciatica with S1 Topography

Correct Answer: B. The ankle reflex is diminished or abolished

Explanation: In a lumbosciatica disc with S1 involvement, the most characteristic finding is a diminished or abolished ankle reflex (B). This is due to the involvement of the S1 nerve root which is responsible for the ankle reflex. Hypoesthesia in S1 typically affects the lateral aspect of the foot, not the dorsal surface of the big toe (A). Walking on heels is more challenging with L5 nerve root involvement (C). Radiological narrowing of the L5-S1 disc is not always present (D), and an electromyogram may show abnormalities but is not constantly disturbed (E).

  1. Topic: Elective Hernia Repair in the Context of Cardiovascular History

Correct Answer: C. Jugular venous distension

Explanation: Jugular venous distension (C) is a sign of right-sided heart failure or fluid overload and indicates potential cardiovascular instability, which could complicate surgery. This would be a valid reason to delay elective hernia repair. Recent coronary artery bypass surgery (A) is a concern but may not be an immediate reason to delay unless there are specific complications. A history of smoking (B), hypertension (D), and hyperlipidemia (E) are risk factors but are not immediate contraindications for delaying surgery in this context.

  1. Topic: Post-Celiotomy Intestinal Motility

Correct Answer: B. To the stomach in 24 hours, the small intestine in 4 hours, and the colon in 3 days

Explanation: Normal intestinal motility typically resumes in the following order after celiotomy: the small intestine usually recovers within 4 hours, the stomach within 24 hours, and the colon generally returns to normal function within 3 days. This timeline reflects the typical postoperative recovery pattern. Immediate resumption (E) and longer durations (C) are less common and do not align with typical postoperative recovery.

  1. Topic: Preoperative Characteristics and Postoperative Ischemia

Correct Answer: C. Dyspnea on exertion

Explanation: Dyspnea on exertion indicates underlying cardiac or pulmonary issues that may increase the risk of postoperative ischemia. Other preoperative characteristics such as angina pectoris (A) or frequent premature ventricular contractions (B) are significant but less directly indicative of immediate postoperative ischemia risk. Tricuspid insufficiency (D) and age over 60 (E) may be relevant but are not as direct indicators of the risk as dyspnea on exertion.

  1. Topic: Blood Transfusion and Clotting

Correct Answer: D. Transfusion through Ringer’s lactate solution

Explanation:
Clotting of transfused blood is typically associated with the use of incompatible blood types, such as ABO or Rh incompatibility (A and C), but Ringer’s lactate solution (D) can cause clotting due to its calcium content, which can activate coagulation. Minor blood group incompatibility (B) is less likely to cause immediate clotting issues. Transfusion through 5% glucose and water (E) does not generally lead to clotting.

  1. Topic: Wound Healing

Correct Answer: E. Rather in wound healing, type 1 collagen is predominant

Explanation: In wound healing, type 1 collagen is the predominant type, especially in the later stages of healing, as it provides strength and structural support. Collagen content generally increases over several weeks, not just one week (A). Monocytes play a broader role in wound healing beyond just phagocytosis, including modulating inflammation and tissue repair (B and D). Fibroblasts appear somewhat later, typically after 48-72 hours (C).

  1. Topic: Wound Contraction

Correct Answer: D. It is based on specialized fibroblasts which contain actin myofilaments

Explanation: Wound contraction is a key process in wound healing, primarily driven by specialized fibroblasts called myofibroblasts, which contain actin myofilaments that help pull the edges of the wound together (D). This process is not the primary mechanism in sutured or stapled wounds, as they primarily heal through primary intention (A). Bacterial colonization can indeed hinder the healing process, but it is not a defining factor for contraction (B). Contraction typically contributes less than 50% to the total wound size reduction (C). The reduction percentage or skin adhesion details mentioned are not standard or relevant measures for contraction (E).

  1. Topic: Management of Thigh Injury with Compartment Syndrome

Correct Answer: B. Immediate exploration and repair

Explanation: In a case of thigh injury with paresthesias, weak pulses, and inability to move the foot, there is a high suspicion of vascular injury and possible compartment syndrome (B). Immediate exploration and repair are crucial to address potential vascular damage and prevent complications such as limb ischemia. Angiography (A) can be useful for diagnostic purposes but is not the immediate intervention required. Anterior compartment fasciotomy (C) is indicated if compartment syndrome is confirmed, but the priority is to explore and repair the injury first. Observation (D) and local exploration (E) are inadequate for addressing the potential severity of the injury and its complications.

  1. Topic: Postoperative Mortality After Heart Transplantation

Correct Answer: C. Accelerated graft arteriosclerosis

Explanation: After the first year post-heart transplantation, accelerated graft arteriosclerosis becomes the most common cause of death (C). This condition involves progressive narrowing and hardening of the transplanted heart’s arteries, leading to graft failure. While infections (A), arrhythmias (B), acute rejection episodes (D), and cancer (E) are significant concerns, their prevalence in causing death decreases after the initial postoperative period. Accelerated graft arteriosclerosis emerges as the leading cause due to its chronic and progressive nature, impacting long-term survival.

  1. Topic: Major Histocompatibility Complex (MHC) Proteins

Correct Answer: D. MHC class 1 proteins act as major targets for antibody-mediated allograft rejection and are detected by cross-match techniques

Explanation: MHC class 1 proteins are indeed major targets for antibody-mediated allograft rejection and are detected by cross-match techniques (D). These proteins are present on almost all nucleated cells and are crucial for presenting endogenous antigens to cytotoxic T cells. MHC class 2 proteins, in contrast, are primarily expressed by antigen-presenting cells (B). They are encoded by different loci (e.g., HLA-DP, HLA-DQ, HLA-DR) and are involved in presenting exogenous antigens to helper T cells (E). The other statements are incorrect as MHC class 1 proteins are not restricted to mononuclear cells (A), MHC class 2 proteins are not encoded by the HLA locus mentioned (C), and MHC class 2 proteins are not the main targets of cross-match techniques (D).

  1. Topic: Treatment of Chronic Ulcer

Correct Answer: C. Billroth 1 type partial gastrectomy

Explanation:
When a gastric ulcer does not improve with standard medical treatment such as H2 blockers and antacids, and especially if there is no improvement after 6 weeks, surgical intervention may be necessary. Billroth 1 type partial gastrectomy is a procedure that involves removing part of the stomach and is suitable for ulcers that are resistant to medical therapy (C). Local excision of the ulcer (B) is generally not sufficient as it doesn’t address potential underlying issues. Resuming the trial medical treatment (A) is not appropriate if the ulcer remains unchanged after adequate treatment. Vagotomy with pyloroplasty (E) or without (D) might be considered, but these are generally part of a more complex surgical approach, often used when ulcer is complicated or recurrent despite other interventions.

  1. Topic: Effects of Colon Resection

Correct Answer: B. Patients who undergo major colon resections experience little change in their bowel habits following the operation

Explanation: After major colon resections, patients generally adapt well, experiencing minimal long-term changes in bowel habits due to compensatory mechanisms of the remaining bowel (B). The net absorption of water by the rectum (A) is not well established, and the right colon actually plays a more significant role in water and electrolyte absorption compared to the left colon (C, D). The role of the ileocecal valve in fluid homeostasis is not as prominent as once thought (E).

  1. Topic: Diagnosis of Severe Upper Left Side Pain

Correct Answer: E. Rupture of the spleen

Explanation: The patient’s presentation, including severe upper left side pain, pallor, elevated pulse, tense and rigid abdomen without peristalsis, suggests rupture of the spleen (E). This condition can follow trauma or infection, leading to internal bleeding and abdominal rigidity. Acute pyelonephritis (A) and Fitz Hugh Curtis Syndrome (B) usually have different pain locations and symptoms. Perforated peptic ulcer (C) and peritonitis (D) are possible but less likely given the history and clinical findings.

  1. Topic: Midline Neck Swelling in Newborn

Correct Answer: C. Cystic hygroma

Explanation: A large midline neck swelling in a newborn that transilluminates is most likely a cystic hygroma (C). Cystic hygromas are congenital lymphatic malformations that are typically filled with lymphatic fluid and show transillumination. A branchial cyst (A) is usually lateral and not typically transilluminated. Cervical adenopathy (B) and goiter (D) do not commonly present with transillumination. A thyroglossal cyst (E) is midline but usually does not transilluminate and is less common with a significant swelling involving the tongue.

  1. Topic: Characteristics of Venous Ulcers

Correct Answer: B. They are usually found on the inner side of the leg

Explanation:
Venous ulcers typically occur on the inner side of the leg, particularly around the medial malleolus (B). They are often painful (A), and compression therapy, such as with compression stockings, is a key part of treatment (C). Venous ulcers do not usually require hospitalization (D) and are more common than arterial ulcers (E).

_____________________________________________________________________________________Top of Form

  1. Topic: Management of Acute Dyspnea Post-Trauma

Correct Answer: C. Decompress the right pleural space

Explanation: In the context of acute dyspnea after a car accident with markedly reduced breath sounds on one side, the most immediate concern is a possible tension pneumothorax or pleural effusion, which requires urgent decompression (C). While a chest X-ray (A) can help diagnose the issue, it should not delay treatment. Arterial blood gas analysis (B), pericardiocentesis (D), and intravenous fluids (E) may be important later but are not the immediate priorities if the clinical picture suggests a pleural space issue.

 

  1. Topic: Acute Increase in Intracranial Pressure

Correct Answer: A. Respiratory irregularities

Explanation: An acute increase in intracranial pressure often leads to respiratory irregularities (A) due to pressure on the brainstem, which controls breathing. Lower blood pressure (B) can occur but is less directly indicative than respiratory changes. Tachycardia (C) is not a primary sign of increased intracranial pressure; instead, it may occur in other conditions. Retinal hemorrhage (D) and compression of the fifth cranial nerve (E) are not as directly associated with acute intracranial pressure increases.

  1. Topic: Obstructive Jaundice in Chronic Pancreatitis

Correct Answer: D. Narrowing of the bile duct

Explanation: In patients with chronic pancreatitis, narrowing of the bile duct (D) is a common cause of obstructive jaundice due to inflammation and fibrosis affecting the bile duct. Pseudocyst formation (A), gallstones (B), and pancreatic head tumors (C) can also cause obstructive jaundice but are less common in this context. Cholangiocarcinoma (E) is a potential cause but is less directly related to chronic pancreatitis compared to bile duct narrowing.

  1. Topic: Malignancy with Specific Symptoms

Correct Answer: A. Gastric carcinoma

Explanation: The symptoms described—difficulty swallowing, vomiting, weight loss, melaena, supraclavicular lymphadenopathy, and a hyperpigmented velvety patch (aurora)—are indicative of gastric carcinoma (A). This condition often presents with systemic symptoms and associated signs like the Virchow’s node (supraclavicular lymphadenopathy) and Leser-Trélat sign (hyperpigmented patch). While esophageal (C) and pancreatic (D) carcinomas can also present with dysphagia and weight loss, the specific combination of symptoms, including melaena and cutaneous findings, strongly points to gastric carcinoma.

  1. Topic: Sliding Inguinal Hernia

Correct Answer: D. Can lead to intestinal damage if not diagnosed

Explanation: A sliding inguinal hernia, where the hernia sac includes a portion of the abdominal wall structures, such as the bladder or colon, can indeed lead to intestinal damage if not identified and managed properly (D). The risk of intestinal damage arises because of the potential for incarceration or strangulation of the involved organs. While sliding inguinal hernias can sometimes be challenging to diagnose pre-operatively (A), they can have serious consequences, making early detection and appropriate surgical intervention crucial.

  1. Topic: Bacterial Septicemia in Splenectomized Patients

Correct Answer: D. Pneumococcus

Explanation: Splenectomized patients are particularly vulnerable to severe infections due to the loss of their spleen’s role in filtering pathogens and producing antibodies. Pneumococcus (D) is especially threatening in these individuals as it can cause severe pneumococcal infections, which are more common and more serious in the absence of a functioning spleen. While Staphylococcus aureus (A), Colibacillus (B), Salmonella (C), and Listeria (E) can also cause infections, pneumococcus is the most notably dangerous due to its propensity to cause rapid and severe illness in this high-risk population.

  1. Topic: Upper Gastrointestinal Bleeding

Correct Answer: D. Diagnostic endoscopy is best done within the first 24 hours

Explanation: In cases of upper gastrointestinal bleeding, performing diagnostic endoscopy within the first 24 hours (D) is crucial for identifying the source of bleeding and providing timely intervention. This early endoscopy improves the chances of visualizing and managing the bleeding point effectively. The bleeding point is typically located above the ligament of Treitz (A), and while angiodysplastic lesions (B) are a cause, they are not the most common. Hematemesis (C) is not always present; patients may have melena or occult bleeding. Selective mesenteric angiography (E) is more often used for cases where endoscopy is not successful or in hemodynamically unstable patients.

  1. Topic: Long-term Complications of Gastrectomy

Correct Answer: E. Vitamin B12 deficiency

Explanation: Vitamin B12 deficiency (E) is actually a recognized long-term complication of total or partial gastrectomy, as the absorption of this vitamin is impaired following surgery. This is due to the removal of parts of the stomach, which reduces intrinsic factor production essential for Vitamin B12 absorption. Gastric cancer (A) is a potential long-term risk, particularly if residual gastric tissue is present. Obstruction (B) can occur due to adhesions or other changes in anatomy post-surgery. Folate deficiency (C) and iron deficiency (D) are also common due to changes in digestive and absorptive processes.

  1. Topic: Urinary Incontinence

Correct Answer: C. It can be helped with drug treatment

Explanation: Drug treatment can be effective in managing urinary incontinence, particularly in cases of urge incontinence or overactive bladder. For the patient described, who experiences difficulty affording diapers and has a history of urinary leakage and smoker’s cough, pharmacological interventions could provide relief. Pelvic floor weakness (B) is a common cause of urinary incontinence, but it is not the only factor, and treatment often starts with conservative measures such as pelvic floor exercises before considering surgery. Most women with urinary incontinence do not seek medical care (A), and bladder instability (D) can be present even with small, frequent, and urgent urination. Surgical intervention (E) is usually considered after other treatments fail, not as an immediate solution.

  1. Topic: Esophageal Symptoms and Diagnosis

Correct Answer: B. Investigation by barium swallow is indicated

Explanation: Given the patient’s difficulty swallowing both solids and liquids, weight loss, and the presence of back tarry stools (indicating possible gastrointestinal bleeding), a barium swallow study is an appropriate initial diagnostic tool to evaluate esophageal pathology. This test helps visualize structural abnormalities and motility issues. The prognosis (A) would be uncertain without further diagnostic clarification. The patient’s symptoms are more indicative of structural or obstructive issues rather than ulcerative peptic disease (C) or achalasia (D). Barium swallow is crucial for determining the underlying cause, such as esophageal cancer or strictures, rather than assuming other diagnoses or outcomes (E).

  1. Topic: Leriche Syndrome (Atherosclerotic Occlusive Disease of the Abdominal Aortic Bifurcation)

Correct Answer: A. Claudication of the buttock and thigh

Explanation: Leriche syndrome, which involves occlusion at the level of the abdominal aortic bifurcation, typically presents with claudication of the buttocks and thighs due to impaired blood flow to these regions during physical activity. Symptoms like causalgia (B) are less characteristic for this condition. Retrograde ejaculation (C) and gangrene of the toes (D) are not directly associated with Leriche syndrome but may relate to more distal vascular issues. Redness dependent on the feet (E) is also not a typical feature of this syndrome. The primary symptom is claudication in the buttocks and thighs due to reduced arterial flow.

  1. Topic: Management of Renal Stones

Correct Answer: A. Conservative treatment including hydration and analgesics will not give a satisfactory result

Explanation: For a stone of 1.5 cm in diameter, conservative treatment alone (hydration and analgesics) is often insufficient due to the size of the stone, which may not pass spontaneously. Serial radiography (B) is used to monitor stone progress but is not always necessary unless complications arise. Urinalysis (C) frequently shows microhematuria but is not diagnostic on its own. While acute episodes are managed, recurrence is common without addressing underlying causes (D). High blood urea and serum creatinine (E) might be present due to obstructive uropathy but are not directly related to the effectiveness of initial conservative treatment.

  1. Topic: Management of Urethral Rupture in Pelvic Fractures

Correct Answer: E. Immediate placement of a suprapubic cystostomy probe

Explanation: In cases of pelvic fracture with urethral rupture, especially when the rupture is located above the urogenital diaphragm, immediate placement of a suprapubic cystostomy (E) is necessary to divert urine and prevent further injury to the urethra. This approach avoids complications that may arise from attempting to place a Foley catheter through the urethra (B) in the presence of a rupture. Immediate surgical reconstruction (C) is usually not done at the initial stage due to the need for patient stabilization. Percutaneous nephrostomy (A) is not the first choice for managing urethral injuries. Exploration of the pelvis (D) may be necessary but is secondary to managing the urethral injury.

  1. Topic: Brain Contusions

Correct Answer: B. They can occur on the opposite side of the point of cranial impact

Explanation: Brain contusions often occur on the opposite side of the impact, known as a “coup-contrecoup” injury (B). This phenomenon happens due to the brain’s motion within the skull. Contusions are more commonly seen in the frontal and temporal lobes rather than the occipital lobes (A). They are typically accompanied by parenchymal bleeding (C) and can be exacerbated by anticoagulants (D). Anticonvulsant drugs (E) are used to manage seizures that might occur post-contusion, making them relevant in early management.

  1. Topic: Acute Abdominal Pain

Correct Answer: B. Acute mesenteric ischemia

Explanation: Acute mesenteric ischemia (B) is a condition where blood flow to the intestines is critically reduced, leading to potential bowel necrosis. This situation requires immediate surgical intervention to restore blood flow and prevent extensive damage. In contrast, acute pancreatitis (A), acute diverticulitis (C), acute cholecystitis (D), and acute Crohn’s disease (E) can often be managed initially with conservative treatments or less urgent surgical approaches depending on the severity and response to medical management.

  1. Topic: Neck Mass Diagnosis

Correct Answer: A. Branchial cyst

Explanation: A branchial cyst (A) typically presents as a mass located along the anterior border of the sternocleidomastoid muscle, which is consistent with the description. These cysts are remnants of the branchial apparatus and are often found in this area. Cervical adenopathy (B), goiter (C), submandibular adenopathy (D), and thyroglossal cyst (E) have different common locations and characteristics that do not match the typical presentation of a branchial cyst.

  1. Topic: Ascitic Fluid Infection

Correct Answer: C. Presence of a germ in culture

Explanation: The presence of a germ in culture (C) is definitive for confirming an infection in ascitic fluid. This is the most specific indicator as it directly identifies the causative pathogen. Elevated protein levels (A), neutrophil count (B), high amylase levels (D), and low pH (E) can indicate infection or other abnormalities but are not as definitive as a positive culture.

  1. Topic: Zenker’s Diverticulum

Correct Answer: B. Located at the level of the crico-pharyngeal space

Explanation: Zenker’s diverticulum (B) is typically located at the level of the crico-pharyngeal space, specifically in the area where the pharynx meets the esophagus, just above the cricopharyngeal muscle. It is not generally associated with gastroesophageal reflux (A) nor commonly seen in young adults (E). It does not have a high risk of cancerous degeneration (C) and is not related to adenopathies (D).

 

GYNAECOLOGY

  1. Topic: Estrogen-Progestogen Strip

Correct Answer: B. Of a combined mini-dose

Explanation: A combined mini-dose pill (B) refers to an oral contraceptive that contains a low dose of both estrogen and progestogen. In this case, the 21 identical pills each containing 30 micrograms of ethinyl estradiol and a progestogen qualify as a combined mini-dose pill. It is not a classic pill (A), micropill (D), sequential (C), or biphasic mini-dose (E).

  1. Topic: Maternal-Fetal Blood Immunization

Correct Answer: C. Mother group O rhesus – negative indirect Coombs test child group 0 Rh+

Explanation: Anti-D Rh immunoglobulin is administered to prevent hemolytic disease of the newborn when there is a risk of Rh incompatibility. This is necessary when an Rh-negative mother (O rhesus -) has an Rh-positive child (0 Rh+), particularly if the mother has a negative indirect Coombs test, indicating she has not yet developed antibodies. In option C, the mother is Rh-negative with a negative Coombs test and the child is Rh-positive, making this the correct scenario for administering anti-D immunoglobulin to prevent sensitization.

  1. Topic: Tumor of the Breast

Correct Answer: B. Cytopunction

Explanation: When a woman presents with a breast lump, especially in the upper outer quadrant, and a mammogram has been performed, cytopunction (fine needle aspiration or biopsy) is the essential next step. This procedure allows for the extraction of cells or tissue from the lump for cytological examination, helping to confirm the diagnosis and guide further treatment. While other tests like ultrasound can provide additional imaging, cytopunction is crucial for a definitive diagnosis.

  1. Topic: Paget’s Disease

Correct Answer: C. Glomerular nephropathy

Explanation: Paget’s disease primarily affects the bones, leading to complications such as sarcomatous degeneration (a type of bone cancer), spontaneous fractures due to weakened bone structure, congestive heart failure from the increased cardiac workload, and spinal compression due to bone deformities. However, glomerular nephropathy (a kidney condition) is not associated with Paget’s disease, making it the incorrect answer in this context.

_____________________________________________________________________________________Top of Form

  1. Topic: Infectious Disease During Pregnancy

Correct Answer: E. Specific immunoglobulin and hepatitis B vaccination

Explanation: Newborns born to mothers who are positive for both HBsAg and HBeAg have a high risk of hepatitis B infection. The recommended preventive treatment is the administration of specific hepatitis B immunoglobulin immediately after birth, along with the initiation of the hepatitis B vaccination series. This combined approach provides immediate passive immunity and helps the newborn develop active immunity against the virus, significantly reducing the risk of chronic hepatitis B infection.

  1. Topic: Ovulation and Menstrual Cycle

Correct Answer: C. 18 days

Explanation: For a woman with a regular menstrual cycle of 32 days, the luteal phase (post-ovulatory phase) typically lasts 14 days. To calculate the date of ovulation, the luteal phase length is subtracted from the total cycle length. Therefore, the pre-ovulatory phase would be 32 – 14 = 18 days. This means that ovulation occurs approximately 18 days before the next expected period.

  1. Topic: Menopause Symptoms

Correct Answer: D. Breast tension

Explanation: Menopause is commonly associated with symptoms such as paresthesia of the extremities, insomnia, vaginal dryness, and sweating due to hormonal changes. However, breast tension is not typically a manifestation of menopause; in fact, it is more often associated with the menstrual cycle or hormonal fluctuations prior to menopause.

  1. Topic: Vulvar Lesions

Correct Answer: C. Genital herpes

Explanation: The presence of clustered blisters on the labia majora, some of which are broken, along with painful symptoms, ipsilateral inguinal adenopathies, and fever (38°C) are classic signs of genital herpes. This condition is caused by the herpes simplex virus and is characterized by painful blisters and ulcers in the genital area. The other conditions listed do not typically present with these specific symptoms.

 

  1. Topic: Breast Tumor Classification

Correct Answer: B. T1 N1 MX

Explanation: In this scenario, the tumor is 2 cm in diameter, which places it in the T1 category (tumor ≤2 cm). The presence of clinically immobile ipsilateral axillary lymphadenopathy suggests N1 (regional lymph nodes involved). Since no metastatic assessment has been conducted, the MX classification is used, indicating that the presence of distant metastasis cannot be evaluated. Thus, the tumor is classified as T1 N1 MX.

  1. Topic: Female Genital Trichomoniasis

Correct Answer: B. It is easily diagnosed on smears taken from the cervix

Explanation: Trichomoniasis is a sexually transmitted infection (STI) that is typically diagnosed using a wet mount or culture from vaginal secretions rather than cervical smears. While the other statements about trichomoniasis being a venereal disease, responding well to Flagyl treatment, requiring partner treatment, and possibly being associated with vaginal candidiasis are correct, diagnosis from cervical smears is not typically accurate, making statement B incorrect.

  1. Topic: Acute Salpingitis

Correct Answer: C. ABDE

Explanation: Acute salpingitis, which is an inflammation of the fallopian tubes, can lead to several complications and remote effects, including: Dyspareunia (A): Painful intercourse. Abdominal pain (B): Common symptom due to inflammation. Ectopic pregnancy (D): Resulting from scarring and adhesions that obstruct the fallopian tube. Tubal sterility (E): Due to scarring that can block the fallopian tubes, affecting fertility. However, chronic urinary tract infections (C) are not directly caused by acute salpingitis. Therefore, the correct set of consequences includes A, B, D, and E.

  1. Topic: Kleihauer Test

Correct Answer: C. Look for the presence of fetal red blood cells in the maternal blood

Explanation: The Kleihauer test is used to detect and quantify fetal red blood cells in the maternal circulation. This test is particularly useful in assessing the extent of fetal-maternal hemorrhage, which can help in managing cases such as Rh incompatibility and in determining the need for additional doses of anti-D immunoglobulin.

  1. Topic: Diagnosis of Endometrial Cancer

Correct Answer: E. Biopsy E- curettage

Explanation: A biopsy with curettage is the most definitive method for diagnosing endometrial cancer. This procedure involves obtaining a tissue sample from the endometrium for histological examination, which can confirm the presence of cancerous cells with high accuracy. Other methods like ultrasound or endometrial smear can suggest abnormalities but are not as definitive as biopsy and curettage.

  1. Topic: Breast Cancer Screening

Correct Answer: B. Mammography

Explanation: Mammography is the most sensitive and widely used method for breast cancer screening. It can detect early signs of cancer, such as microcalcifications, before they become palpable. While other methods like ultrasound can be useful in specific cases, mammography remains the gold standard for routine screening due to its ability to detect small, asymptomatic tumors.

  1. Topic: Confirming Pregnancy Viability

Correct Answer: E. Ultrasound

Explanation: Ultrasound is the most reliable method to confirm the viability of a pregnancy, especially in cases of amenorrhea with bleeding. It allows direct visualization of the gestational sac, fetal heartbeat, and the overall development of the embryo or fetus, providing clear evidence of whether the pregnancy is progressing normally. Other tests, such as HCG levels, can indicate pregnancy but do not confirm its viability as effectively as an ultrasound.

  1. Topic: Cervical Cancer and Infectious Agents

Correct Answer: D. Papillomavirus

Explanation: Human papillomavirus (HPV) is the primary infectious agent linked to the development of cervical cancer. Persistent infection with high-risk HPV strains, particularly types 16 and 18, is a significant factor in the pathogenesis of cervical cancer. Other listed pathogens, such as Trichomonas, Gonococcus, Chlamydia, and Mycoplasma, are associated with other genital infections but are not directly involved in the genesis of cervical cancer.

  1. Topic: Postmenopausal Screening

Correct Answer: D. A mammogram

Explanation: A mammogram is routinely recommended for postmenopausal women as part of breast cancer screening. Early detection through mammography can significantly improve outcomes in breast cancer treatment. While bone density assessments, such as a lumbar spine X-ray, are also important, they are typically used to diagnose osteoporosis rather than being a systematic screening tool for all postmenopausal women. Hormone level testing like FSH or estrogen dosage and procedures like hysterography are not standard screenings for this population.

  1. Topic: Stage IB Cervical Cancer

Correct Answer: C. Micro invasive cancer

Explanation: Stage IB cervical cancer is characterized by a visible lesion that is confined to the cervix, including both squamous cell carcinoma and adenocarcinoma. However, it does not include microinvasive cancer, which is classified under Stage IA. Microinvasive cancer refers to very early-stage cancer where invasion is minimal, typically less than 5 mm deep and less than 7 mm in horizontal spread. Stage IB cancers can show more extensive invasion, but still limited to the cervix, with potential lymphatic involvement.

 

  1. Topic: Fibroids and Pregnancy

Correct Answer: B. If there is a fibroid in the 3rd trimester of pregnancy a myomectomy must be performed

Explanation: Myomectomy, the surgical removal of fibroids, is generally not recommended during the third trimester of pregnancy due to the high risk of complications such as bleeding and preterm labor. The preferred approach is to manage the pregnancy conservatively and plan for the fibroid’s removal after childbirth if necessary. The other statements regarding fibroids interfering with early pregnancy diagnosis, causing aseptic necrobiosis, leading to abnormal fetal presentations, and potentially obstructing childbirth are accurate.

  1. Topic: Newborn Care for HBsAg and HBeAg Positive Mother

Correct Answer: E. Serovaccination

Explanation: For a newborn born to an HBsAg and HBeAg positive mother, the recommended care involves serovaccination, which includes the administration of both specific hepatitis B immunoglobulins (HBIG) and the hepatitis B vaccine. This combined approach provides immediate passive immunity from the immunoglobulins and active immunity through the vaccine, significantly reducing the risk of the newborn developing hepatitis B infection.

  1. Topic: Risk Factors for Upper Genital Tract Infection

Correct Answer: B. Increased the risk by taking a progestogen

Explanation: The correct statement is that taking a progestogen does not increase the risk of upper genital tract infections. In fact, progestogens may help thicken cervical mucus, potentially reducing the risk of infection. The other factors mentioned, such as age, nulliparity, IUD use, and local contraceptives, are correctly identified as influencing the risk of upper genital tract infections.

  1. Topic: Ovarian Cancer Symptoms

Correct Answer: C. An increase in the volume of the abdomen

Explanation: The most common symptom revealing ovarian cancer is an increase in the volume of the abdomen. This often results from ascites or a growing mass. Other symptoms like metrorrhagia, signs of rectal or bladder compression, and supraclavicular lymphadenopathy can occur, but they are less common as initial presenting signs.

  1. Topic: Pregnancy and Blood Pressure

Correct Answer: B. Blood pressure rises during normal pregnancy

Explanation: During normal pregnancy, blood pressure typically decreases due to increased plasma volume and decreased peripheral resistance. An increase in blood pressure during pregnancy is not normal and may indicate conditions such as preeclampsia. The other statements are accurate: plasma volume increases, peripheral resistance decreases, there is a link between hypertension and fetal growth disorders, and a salt-free diet is not generally recommended for managing pregnancy-induced hypertension.

  1. Topic: Monitoring Treated Cervical Cancer

Correct Answer: D. Regular surveillance smears

Explanation: For monitoring treated cervical cancer, regular surveillance smears (Pap smears) are crucial to detect any residual or recurrent disease. This method allows for early identification of abnormalities that might indicate a return of cancer. While clinical examination is important, surveillance smears provide specific information about cervical cell changes. Beta2-macroglobulin and alpha-fetoprotein are not typically used for cervical cancer monitoring, and bone scans are used primarily to assess metastatic spread rather than for routine follow-up.

  1. Topic: Complications of Estrogen Deficiency in Menopause

Correct Answer: E. AC (Vaginal atrophy and Breast involution)

Explanation: Estrogen deficiency during menopause can lead to vaginal atrophy, which causes thinning and dryness of the vaginal walls. Breast involution, or the reduction of breast tissue, also occurs due to decreased estrogen levels. Endometrial hyperplasia and increased cervical mucus secretion are more associated with other hormonal imbalances or conditions, rather than directly with estrogen deficiency.

  1. Topic: Fundal Height Measurement

Correct Answer: B. Fetal hypotrophy

Explanation: A fundal height of 26 cm at 34 weeks of amenorrhea is below the expected range for this stage of pregnancy, which is typically around 30-32 cm. This discrepancy suggests fetal hypotrophy (growth restriction) rather than a normal pregnancy. Twin pregnancies and hydramnios generally cause increased fundal height, while hypertrophy of the placenta might cause height changes but is less commonly diagnosed solely by fundal measurement.

  1. Topic: Suspected Ectopic Pregnancy

Correct Answer: C. Ask for a beta HCG assay and an ultrasound

Explanation: When clinical examination is inconclusive in a suspected ectopic pregnancy, a beta HCG assay and ultrasound are essential to confirm the diagnosis. These tests help assess the levels of pregnancy hormones and visualize the location of the pregnancy. Laparoscopy or immediate surgery is not indicated unless there are signs of acute complications or the tests confirm an ectopic pregnancy.

  1. Topic: Presentation Engagement in Labor

Correct Answer: B. Has crossed the upper strait

Explanation: Engagement of a presentation refers to the fetal presenting part (typically the head) entering and descending through the pelvic inlet (upper strait). Once the largest diameter of the presenting part has crossed the upper strait, it is considered engaged. This does not mean it has passed through the entire pelvis or reached the vulva; that would indicate further progress in labor.

  1. Topic: Diagnosis of Genital Prolapse

Correct Answer: A. On clinical examination

Explanation: Genital prolapse is primarily diagnosed through a clinical examination where the physician can observe and assess the presence of prolapse. Although imaging techniques like ultrasound and hysterography can be used for further evaluation, the initial diagnosis is typically made during a physical examination.

  1. Topic: Low Insertion of the Placenta

Correct Answer: E. Ultrasound can make the diagnosis at 35 weeks

Explanation: A low insertion of the placenta, also known as placenta previa, is typically diagnosed using ultrasound imaging, which can accurately assess the placental position. This diagnosis is usually confirmed around 35 weeks of gestation. While other complications related to low placental insertion, such as bleeding, can occur, the definitive diagnosis is made through ultrasound.

  1. Topic: Treatment of Threatened Preterm Labor

Correct Answer: C. Beta blocker

Explanation: In the treatment of threatened preterm labor, common medications include progesterone (to support the pregnancy), aspirin (to reduce inflammation and improve blood flow), indomethacin (a nonsteroidal anti-inflammatory drug that can inhibit contractions), and salbutamol (a beta-agonist that can help relax the uterus). However, beta blockers are not typically used for this purpose and are not effective in preventing preterm labor.

  1. Topic: Ectopic Pregnancy

Correct Answer: B. It is favored by taking estrogen-progestogen contraceptive pills

Explanation: Ectopic pregnancy is more common in women with a history of tubal infertility treatments and can be associated with conditions that affect the fallopian tubes. A normal pelvic ultrasound does not completely rule out ectopic pregnancy, and a beta HCG plasma level below 5 IU can help in excluding it. Isthmic tubal pregnancies are indeed at a higher risk of catastrophic rupture. However, estrogen-progestogen contraceptive pills are not typically associated with an increased risk of ectopic pregnancy.

_____________________________________________________________________________________Top of Form

  1. Topic: Asymptomatic Bacteriuria in Pregnancy

Correct Answer: A. Must be treated with appropriate anti-infectives

Explanation: Asymptomatic bacteriuria in pregnant women, even if it does not present symptoms, should be treated with appropriate antibiotics. This is because untreated bacteriuria can lead to complications such as pyelonephritis and adverse outcomes for both the mother and the fetus. Urography and cystography are not typically required in this scenario, and indwelling catheters and bed rest are not standard treatments for asymptomatic bacteriuria.

  1. Topic: Fetal Complications from Urinary Tract Infections in Pregnancy

Correct Answer: B. Prematurity

Explanation: Urinary tract infections (UTIs) during pregnancy can lead to several fetal complications, with prematurity being one of the most common. UTIs can cause preterm labor, leading to premature birth. Other complications like intrauterine fetal death, retroplacental hematoma, and macrosomia are less directly linked to UTIs compared to prematurity.

  1. Topic: Monitoring Pregnancy with Vascular-Renal Risk

Correct Answer: C. Uremia

Explanation: In pregnancies with vascular-renal risk, monitoring typically involves assessing serum creatinine, uricemia, 24-hour proteinuria, and blood pressure. Uremia is not commonly used as a routine monitoring parameter in this context; instead, serum creatinine and other markers provide more immediate information on renal function.

  1. Topic: Seizures in Pregnant Women with Hypertension

Correct Answer: D. Eclampsia

Explanation: Epileptic seizures in a pregnant woman at the 9th month, especially with hypertension, most commonly suggest eclampsia. Eclampsia is a severe complication of preeclampsia, characterized by seizures in addition to high blood pressure. Other conditions, like cerebral hemorrhage or thrombosis, could also cause seizures but are less directly linked to the combination of late pregnancy and hypertension.

  1. Topic: Assessing Prognosis of Toxaemia of Pregnancy

Correct Answer: D. Uricemia

Explanation: Uricemia (elevated uric acid levels) is a key laboratory test used to assess the prognosis of toxaemia of pregnancy, which includes preeclampsia and eclampsia. Elevated uric acid levels are associated with worsening of the condition and can help in evaluating the severity and prognosis. While creatinine levels can also be important, uricemia is more specifically linked to the progression of toxaemia.

  1. Topic: Most Common Cause of Spontaneous Abortion in the 1st Trimester

Correct Answer: E. Chromosomal abnormalities of the egg

Explanation: Chromosomal abnormalities of the egg are the most common cause of spontaneous abortion in the 1st trimester of pregnancy. These abnormalities often lead to non-viable pregnancies, resulting in early miscarriage. While hormonal causes, uterine malformations, and fibroids can contribute to pregnancy loss, chromosomal abnormalities are the leading factor in early pregnancy loss.

  1. Topic: Risk Factors for Ectopic Pregnancy

Correct Answer: B. BA ECB

Explanation: The risk factors for ectopic pregnancy include:Progestogen-only pill (A): This method is not a significant risk factor for ectopic pregnancy. History of salpingitis (B): Previous infection of the fallopian tubes increases the risk of ectopic pregnancy. History of pelvic surgery (C): Prior pelvic surgery can lead to scarring and increase the risk of ectopic pregnancy. Ovarian cyst (D): Not a risk factor for ectopic pregnancy. Intrauterine device (IUD) (E): Although it primarily prevents pregnancy, if pregnancy occurs with an IUD in place, there is an increased risk of ectopic pregnancy. Thus, the correct risk factors are B, C, and E.

  1. Topic: Causes of Ectopic Pregnancy (GEU)

Correct Answer: A. AA BDE

Explanation: The correct causes of ectopic pregnancy (GEU) include: Sequelae of salpingitis (A): Inflammation of the fallopian tubes can cause scarring, leading to an increased risk of ectopic pregnancy. Tubal endometriosis (B): Endometriosis affecting the fallopian tubes can interfere with normal egg movement, increasing the risk of ectopic pregnancy. Unilateral tubal stenosis (D): The narrowing of one fallopian tube can prevent the egg from reaching the uterus, leading to an ectopic pregnancy. Prolonged egg migration (E): If the egg takes too long to travel through the fallopian tube, it may implant outside the uterus. Thus, the correct answers are A, B, D, and E.

  1. Topic: Contraindications to Contraception

Correct Answer: D. Endometrial hyperplasia

Explanation: Endometrial hyperplasia, a condition characterized by the thickening of the uterine lining, is a contraindication to contraception, particularly hormonal contraceptives. These contraceptives can exacerbate the condition, increasing the risk of developing endometrial cancer. The other conditions listed (lutein cyst, uterine fibroid, endometriosis) are not absolute contraindications to contraception, but their management may require special consideration. Thus, the correct answer is D.

  1. Topic: Contraindications to Estrogen-Progestin

Correct Answer: C. Uterine fibroid

Explanation: Uterine fibroids are not a formal contraindication for the use of estrogen-progestin contraceptives. In fact, hormonal contraceptives can sometimes be used to manage symptoms associated with fibroids, such as heavy menstrual bleeding. On the other hand, malignant tumors of the breast and uterus, thromboembolic accidents, cholestatic hepatitis, and pituitary adenoma are considered contraindications due to the potential for exacerbating these conditions. Therefore, the correct answer is C.

  1. Topic: Contraindications to Oral Contraception by Estrogen-Progestogen

Correct Answer: A. Multiparity

Explanation: Multiparity (having given birth to more than one child) is not a contraindication for the use of estrogen-progestogen oral contraceptives. However, conditions such as arterial hypertension, deep phlebitis, cholestatic hepatitis, and uterine fibroids are considered either absolute or relative contraindications due to the increased risk of complications. Therefore, the correct answer is A.

  1. Topic: Risk Factors for Breast Cancer

Correct Answer: C. BCDE

Explanation: The risk factors for breast cancer include proliferating mastotic lesions (B), lack of breastfeeding (C), late menopause (D), and early first menstruation (E). However, early age at first pregnancy (A) is generally associated with a lower risk of breast cancer, so it is not included among the risk factors. Therefore, the correct answer is C (BCDE).

  1. Topic: Prognosis in Cervical Cancer

Correct Answer: B. Cancer stage

Explanation: The most critical factor for the 5-year survival prognosis after appropriate treatment for cervical cancer is the stage of the cancer at the time of diagnosis. The cancer stage determines the extent of the disease and guides treatment options, making it the most significant predictor of survival outcomes. While other factors like HPV strain, patient age, histological grade, and lymph node involvement are important, they do not outweigh the impact of the cancer stage on prognosis.

  1. Topic: Fetal Red Blood Cell Production

Correct Answer: D. Liver

Explanation: At the 24th week of gestation, the primary site of red blood cell production in the fetus is the liver. The liver is the main hematopoietic organ during this stage of development, taking over from the yolk sac, which is involved earlier in gestation. Eventually, the bone marrow will become the dominant site of red blood cell production as the fetus matures, but at 24 weeks, the liver remains the primary source.

  1. Topic: Lung Maturity Enhancement in Preterm Labor

Correct Answer: B. Betamethasone

Explanation: Betamethasone is a corticosteroid commonly used to enhance fetal lung maturity in cases of preterm labor. Administering this drug accelerates the production of surfactant, a substance that helps keep the airways open in the lungs, significantly reducing the risk of respiratory distress syndrome (RDS) in preterm infants. The other options do not serve this specific function.

  1. Topic: Screening for Fetal Well-being in a High-Risk Pregnancy

Correct Answer: E. Biophysical profile of the fetus

Explanation: The biophysical profile (BPP) is the best test for assessing fetal well-being in a high-risk pregnancy, such as in a woman with type 2 diabetes mellitus and chronic hypertension. The BPP combines an ultrasound evaluation of the fetus with a non-stress test (NST) to assess various parameters like fetal breathing movements, gross body movements, fetal tone, amniotic fluid volume, and fetal heart rate. This comprehensive test provides a better overall assessment of fetal health compared to the other options.

  1. Topic: Risk Factors for Breast Cancer

Correct Answer: E. Having a mother with a history of breast cancer

Explanation: Having a mother with a history of breast cancer is associated with the greatest lifetime risk of developing the disease. This familial link significantly increases the likelihood due to shared genetic factors, such as mutations in the BRCA1 or BRCA2 genes. While factors like early menarche, late menopause, obesity, and age also contribute to breast cancer risk, a family history, particularly a first-degree relative like a mother, is the most significant predictor.

  1. Topic: Types of Breast Cancer

Correct Answer: D. Infiltrating ductal carcinoma

Explanation: Infiltrating ductal carcinoma (IDC) is the most common type of breast cancer. It starts in the ducts of the breast and invades the surrounding tissue. Other types, such as inflammatory carcinoma, lobular carcinoma in situ, lobular infiltrating carcinoma, and ductal carcinoma in situ, are less common or represent different stages or forms of the disease. IDC accounts for the majority of breast cancer cases and is characterized by its ability to spread to other parts of the body.

  1. Topic: Causes of Vaginal Discharge

Correct Answer: B. Trichomonas vaginitis

Explanation: Trichomonas vaginitis typically presents with a vaginal discharge that may cause burning and itching, and the pH of the discharge is often elevated, usually above 4.5. In this case, the pH of 4.5 is on the higher side, suggesting an infection like trichomoniasis.

  1. Topic: Investigation of Symptoms in a Young Woman

Correct Answer: E. Tests for chlamydia research

Explanation: In a young woman presenting with dysuria, vaginal discharge, and pelvic pain, the most appropriate investigation is testing for sexually transmitted infections, such as chlamydia. These symptoms are often associated with infections like chlamydia and gonorrhoea.

  1. Topic: Amenorrhea in a Young Woman After Stopping Birth Control Pills

Correct Answer: A. Dosage of serum prolactin level

Explanation: In a young woman with amenorrhea following discontinuation of birth control pills, and with otherwise good health, evaluating serum prolactin levels is important to rule out hyperprolactinemia, which can cause amenorrhea.

  1. Topic: Metrorrhagia in a Woman on Oral Contraceptives

Correct Answer: D. Dilation and curettage

Explanation: In a woman with metrorrhagia while on oral contraceptives, dilation and curettage (D&C) is often performed to investigate the cause of abnormal bleeding, especially if the bleeding is persistent or unexplained.

View Kamer

FREE
VIEW